Finding a Second Linearly Independent Solution Using Reduction of Order

Click For Summary
The discussion revolves around finding a second linearly independent solution to the differential equation x²y" + xy' + (x² - 1/4) = 0, given that y1 = x^(-1/2)cos(x) is one solution. Participants suggest using the method of reduction of order, starting with a solution of the form y2 = u(x)x^(1/2)cos(x). There is confusion regarding the validity of the provided solution y1, as it may not satisfy the original differential equation. To proceed correctly, it is advised to verify the equation and ensure all terms are appropriately handled to derive the second solution. The conversation emphasizes the importance of correctly applying the reduction of order method to find y2.
Mark Brewer
Messages
38
Reaction score
4

Homework Statement


(Reduction of order) The function y1 = x-1/2cosx is one solution to the differential equation x2y" + xy' + (x2 - 1/4) = 0. Use the method of reduction of order to find another linearly independent solution.

The Attempt at a Solution



I divided x2 to both sides to get the equation into y" + py' + qy = 0

y" + (1/x)y' + ((1 - (1/4x2)) = 0

Using Abel's method

c e-∫(p)dx

= c e-ln x
y2 = -c lnx[/B]

Am I doing this right?
 
Physics news on Phys.org
Mark Brewer said:

Homework Statement


(Reduction of order) The function y1 = x-1/2cosx is one solution to the differential equation x2y" + xy' + (x2 - 1/4) = 0. Use the method of reduction of order to find another linearly independent solution.

The Attempt at a Solution



I divided x2 to both sides to get the equation into y" + py' + qy = 0

y" + (1/x)y' + ((1 - (1/4x2)) = 0

Using Abel's method

c e-∫(p)dx

= c e-ln x
y2 = -c lnx[/B]

Am I doing this right?
Take y2 and plug it back into the original differential equation and see if it is satisfied. :wink:
 
Mark Brewer said:

Homework Statement


(Reduction of order) The function y1 = x-1/2cosx is one solution to the differential equation x2y" + xy' + (x2 - 1/4) = 0. Use the method of reduction of order to find another linearly independent solution.

The Attempt at a Solution



I divided x2 to both sides to get the equation into y" + py' + qy = 0

y" + (1/x)y' + ((1 - (1/4x2)) = 0

Using Abel's method

c e-∫(p)dx

= c e-ln x
y2 = -c lnx[/B]

Am I doing this right?
No, the problem said to use the method of reduction of order.

But first, did you type the differential equation and solution correctly? I ask because the supposed solution doesn't satisfy the given differential equation.
 
Mark Brewer said:

Homework Statement


(Reduction of order) The function y1 = x-1/2cosx is one solution to the differential equation x2y" + xy' + (x2 - 1/4) = 0.
You mean x2y" + xy' + (x2 - 1/4)y = 0, don't you? As Vela pointed out, the given y1 does not satisfy the equation you gave. Perhaps it satisfies this one. I haven't checked.

Use the method of reduction of order to find another linearly independent solution.
So you start by looking for a solution of the form y_2= u(x)x^{1/2}cos(x)
Find the first and second derivatives of that and put them into the equation. If your given function really is a solution to the differential equation, then all terms involving only "u" (as opposed to u' or u'') will cancel leaving a first order equation for v= u'.

3. The Attempt at a Solution

I divided x2 to both sides to get the equation into y" + py' + qy = 0

y" + (1/x)y' + ((1 - (1/4x2)) = 0

Using Abel's method

c e-∫(p)dx

= c e-ln x
y2 = -c lnx
Am I doing this right?
Mark Brewer said:

Homework Statement


(Reduction of order) The function y1 = x-1/2cosx is one solution to the differential equation x2y" + xy' + (x2 - 1/4) = 0. Use the method of reduction of order to find another linearly independent solution.

The Attempt at a Solution



I divided x2 to both sides to get the equation into y" + py' + qy = 0

y" + (1/x)y' + ((1 - (1/4x2)) = 0

Using Abel's method

c e-∫(p)dx

= c e-ln x
y2 = -c lnx[/B]

Am I doing this right?
 
Question: A clock's minute hand has length 4 and its hour hand has length 3. What is the distance between the tips at the moment when it is increasing most rapidly?(Putnam Exam Question) Answer: Making assumption that both the hands moves at constant angular velocities, the answer is ## \sqrt{7} .## But don't you think this assumption is somewhat doubtful and wrong?

Similar threads

Replies
4
Views
1K
  • · Replies 2 ·
Replies
2
Views
2K
  • · Replies 2 ·
Replies
2
Views
1K
Replies
7
Views
2K
  • · Replies 14 ·
Replies
14
Views
1K
  • · Replies 3 ·
Replies
3
Views
2K
  • · Replies 21 ·
Replies
21
Views
2K
  • · Replies 3 ·
Replies
3
Views
2K
  • · Replies 3 ·
Replies
3
Views
2K
  • · Replies 3 ·
Replies
3
Views
1K